Microsoft Math Solver
Λύση
Εξάσκηση
Λήψη
Solve
Practice
Θέματα
Προ-Άλγεβρα
Μέση τιμή
Λειτουργία
Μεγαλύτερος Κοινός Παράγοντας
Ελάχιστο Κοινό Πολλαπλάσιο
Σειρά Εργασιών
Κλάσματα
Μικτά Κλάσματα
Κύρια Παραγοντοποίηση
Εκθέτες
Ρίζες
Άλγεβρα
Συνδυασμός Συναφών Όρων
Επίλυση για μια Μεταβλητή
Παράγοντας
Ανάπτυξη
Αξιολόγηση Κλασμάτων
Γραμμικές Εξισώσεις
Τετραγωνικές Εξισώσεις
Ανισώσεις
Συστήματα Εξισώσεων
Πίνακες
Τριγωνομετρία
Απλοποίηση
Αποτέλεσμα
Γραφήματα
Επίλυση Εξισώσεων
Λογισμός
Παράγωγα
Ολοκληρώματα
Όρια
Αλγεβρική αριθμομηχανή
Αριθμομηχανή τριγωνομετρίας
Αριθμομηχανή λογισμού
Αριθμομηχανή Πινάκων
Λήψη
Θέματα
Προ-Άλγεβρα
Μέση τιμή
Λειτουργία
Μεγαλύτερος Κοινός Παράγοντας
Ελάχιστο Κοινό Πολλαπλάσιο
Σειρά Εργασιών
Κλάσματα
Μικτά Κλάσματα
Κύρια Παραγοντοποίηση
Εκθέτες
Ρίζες
Άλγεβρα
Συνδυασμός Συναφών Όρων
Επίλυση για μια Μεταβλητή
Παράγοντας
Ανάπτυξη
Αξιολόγηση Κλασμάτων
Γραμμικές Εξισώσεις
Τετραγωνικές Εξισώσεις
Ανισώσεις
Συστήματα Εξισώσεων
Πίνακες
Τριγωνομετρία
Απλοποίηση
Αποτέλεσμα
Γραφήματα
Επίλυση Εξισώσεων
Λογισμός
Παράγωγα
Ολοκληρώματα
Όρια
Αλγεβρική αριθμομηχανή
Αριθμομηχανή τριγωνομετρίας
Αριθμομηχανή λογισμού
Αριθμομηχανή Πινάκων
Λύση
άλγεβρα
τριγωνομετρία
στατιστικά
λογισμός
πίνακες
μεταβλητές
λίστα
Λύση ως προς K
K=\frac{4q}{9}
Προβολή βημάτων επίλυσης
Βήματα για την επίλυση γραμμικής εξίσωσης
q = \frac { K ( 2 ) ( 3 ) ^ { 2 } } { 8 }
Υπολογίστε το 3στη δύναμη του 2 και λάβετε 9.
q=\frac{K\times 2\times 9}{8}
Πολλαπλασιάστε 2 και 9 για να λάβετε 18.
q=\frac{K\times 18}{8}
Διαιρέστε το K\times 18 με το 8 για να λάβετε K\times \left(\frac{9}{4}\right).
q=K\times \left(\frac{9}{4}\right)
Κάντε εναλλαγή πλευρών έτσι ώστε όλοι οι μεταβλητοί όροι να βρίσκονται στην αριστερή πλευρά.
K\times \left(\frac{9}{4}\right)=q
Η εξίσωση είναι σε τυπική μορφή.
\frac{9}{4}K=q
Διαιρέστε και τις δύο πλευρές της εξίσωσης με \frac{9}{4}, το οποίο είναι το ίδιο σαν να πολλαπλασιάζατε και τις δύο πλευρές με το αντίστροφο κλάσμα.
\frac{\frac{9}{4}K}{\frac{9}{4}}=\frac{q}{\frac{9}{4}}
Η διαίρεση με το \frac{9}{4} αναιρεί τον πολλαπλασιασμό με το \frac{9}{4}.
K=\frac{q}{\frac{9}{4}}
Διαιρέστε το q με το \frac{9}{4}, πολλαπλασιάζοντας το q με τον αντίστροφο του \frac{9}{4}.
K=\frac{4q}{9}
Λύση ως προς q
q=\frac{9K}{4}
Προβολή βημάτων επίλυσης
Βήματα λύσης
q = \frac { K ( 2 ) ( 3 ) ^ { 2 } } { 8 }
Υπολογίστε το 3στη δύναμη του 2 και λάβετε 9.
q=\frac{K\times 2\times 9}{8}
Πολλαπλασιάστε 2 και 9 για να λάβετε 18.
q=\frac{K\times 18}{8}
Διαιρέστε το K\times 18 με το 8 για να λάβετε K\times \left(\frac{9}{4}\right).
q=K\times \left(\frac{9}{4}\right)
Κουίζ
Linear Equation
5 προβλήματα όπως:
q = \frac { K ( 2 ) ( 3 ) ^ { 2 } } { 8 }
Παρόμοια προβλήματα από την Αναζήτηση στο web
12=2(3)^2/q-3
https://www.tiger-algebra.com/drill/12=2(3)~2/q-3/
12=2(3)2/q-3 One solution was found : q = 6/5 = 1.200 Rearrange: Rearrange the equation by subtracting what is to the right of the equal sign from both sides of the equation : ...
Suppose that q=\frac{2^n+1}{3} is prime then q is the largest factor of \binom{2^n}{2}-1
https://math.stackexchange.com/q/1912522
Since you already have other answers, I will instead offer a small generalization. Suppose p=\frac{k^n +1}{k+1} is a prime number. Then, p is the largest prime factor of \binom{k^n}{2}-1 if k=2j^2 ...
Inverse Laplace Transform Problem
https://math.stackexchange.com/questions/520266/inverse-laplace-transform-problem
You may use the residue theorem. The ILT is \frac{1}{i 2 \pi} \int_{c-i \infty}^{c+i \infty} ds \frac{e^{s t}}{(s^2+1)^3} where c \gt 0. Consider a contour integral in the complex s plane: \frac{1}{i 2 \pi} \oint_C ds \frac{e^{s t}}{(s^2+1)^3} ...
Probability Density Function
https://stats.stackexchange.com/q/374914
The formula means \frac{1}{\sqrt{2\pi}\sigma}\exp\left(-\frac{(x-\mu)^2}{2\sigma^2} \right) After you computed -\frac{(x-\mu)^2}{2\sigma^2} , call it w , we then compute e^w and then ...
Find the structure of \mathbb{Z}[\sqrt[3]{2}]/(4+\sqrt[3]{4})
https://math.stackexchange.com/questions/127872/find-the-structure-of-mathbbz-sqrt32-4-sqrt34
No, it's not (entirely) a coincidence. As an abelian group, A is isomorphic to a direct sum/product of three copies of \mathbb{Z}, A\cong \mathbb{Z}\times\mathbb{Z}\times\mathbb{Z}. The fact ...
Problem understanding a kinematic derivation
https://physics.stackexchange.com/questions/292705/problem-understanding-a-kinematic-derivation
However, the object comes to rest eventually but as we see if v_0>0 then v(t) will never equal zero. Should I add the initial velocity term? Yes, you need to take initial velocity into account ...
Περισσότερα Στοιχεία
Κοινοποίηση
Αντιγραφή
Αντιγράφηκε στο πρόχειρο
q=\frac{K\times 2\times 9}{8}
Υπολογίστε το 3στη δύναμη του 2 και λάβετε 9.
q=\frac{K\times 18}{8}
Πολλαπλασιάστε 2 και 9 για να λάβετε 18.
q=K\times \left(\frac{9}{4}\right)
Διαιρέστε το K\times 18 με το 8 για να λάβετε K\times \left(\frac{9}{4}\right).
K\times \left(\frac{9}{4}\right)=q
Κάντε εναλλαγή πλευρών έτσι ώστε όλοι οι μεταβλητοί όροι να βρίσκονται στην αριστερή πλευρά.
\frac{9}{4}K=q
Η εξίσωση είναι σε τυπική μορφή.
\frac{\frac{9}{4}K}{\frac{9}{4}}=\frac{q}{\frac{9}{4}}
Διαιρέστε και τις δύο πλευρές της εξίσωσης με \frac{9}{4}, το οποίο είναι το ίδιο σαν να πολλαπλασιάζατε και τις δύο πλευρές με το αντίστροφο κλάσμα.
K=\frac{q}{\frac{9}{4}}
Η διαίρεση με το \frac{9}{4} αναιρεί τον πολλαπλασιασμό με το \frac{9}{4}.
K=\frac{4q}{9}
Διαιρέστε το q με το \frac{9}{4}, πολλαπλασιάζοντας το q με τον αντίστροφο του \frac{9}{4}.
q=\frac{K\times 2\times 9}{8}
Υπολογίστε το 3στη δύναμη του 2 και λάβετε 9.
q=\frac{K\times 18}{8}
Πολλαπλασιάστε 2 και 9 για να λάβετε 18.
q=K\times \left(\frac{9}{4}\right)
Διαιρέστε το K\times 18 με το 8 για να λάβετε K\times \left(\frac{9}{4}\right).
Παραδείγματα
Δευτεροβάθμια εξίσωση
{ x } ^ { 2 } - 4 x - 5 = 0
Τριγωνομετρία
4 \sin \theta \cos \theta = 2 \sin \theta
Γραμμική εξίσωση
y = 3x + 4
Αριθμητική
699 * 533
Πίνακας
\left[ \begin{array} { l l } { 2 } & { 3 } \\ { 5 } & { 4 } \end{array} \right] \left[ \begin{array} { l l l } { 2 } & { 0 } & { 3 } \\ { -1 } & { 1 } & { 5 } \end{array} \right]
Σύστημα εξισώσεων
\left. \begin{cases} { 8x+2y = 46 } \\ { 7x+3y = 47 } \end{cases} \right.
Παραγώγιση
\frac { d } { d x } \frac { ( 3 x ^ { 2 } - 2 ) } { ( x - 5 ) }
Ολοκλήρωση
\int _ { 0 } ^ { 1 } x e ^ { - x ^ { 2 } } d x
Όρια
\lim _{x \rightarrow-3} \frac{x^{2}-9}{x^{2}+2 x-3}
Επιστροφή στην αρχή της σελίδας